Difference between revisions of "2024 AMC 10B Problems/Problem 20"
(→Solution 3(focus on restrictions)) |
Pinotation (talk | contribs) (→Solution 2 (Tree Diagram)) |
||
(22 intermediate revisions by 5 users not shown) | |||
Line 6: | Line 6: | ||
<math>\textbf{(A) } 60 \qquad\textbf{(B) } 72 \qquad\textbf{(C) } 90 \qquad\textbf{(D) } 108 \qquad\textbf{(E) } 120</math> | <math>\textbf{(A) } 60 \qquad\textbf{(B) } 72 \qquad\textbf{(C) } 90 \qquad\textbf{(D) } 108 \qquad\textbf{(E) } 120</math> | ||
− | ==Solution 1 | + | ==Solution 1== |
Let <math>A_R, A_L, B_R, B_L, C_R, C_L</math> denote the shoes. | Let <math>A_R, A_L, B_R, B_L, C_R, C_L</math> denote the shoes. | ||
Line 46: | Line 46: | ||
Our final answer is <math>24 + 36 = \boxed{\textbf{(A) } 60}</math> | Our final answer is <math>24 + 36 = \boxed{\textbf{(A) } 60}</math> | ||
− | ==Solution 2 ( | + | ==Solution 2 (Tree Diagram)== |
− | + | Label the pairs \( (L_1, R_1) \), \( (L_2, R_2) \), and \( (L_3, R_3) \). | |
− | + | We now create a tree diagram. [[WLOG]], we say our first tree diagram starts with \( L_1 \). | |
+ | [[File:2024 AMC 10B Q20.png | center | 500px]] | ||
− | ==Solution 3(focus on restrictions)== | + | This tree has 10 branches. |
+ | |||
+ | Notice how we can interchange the first shoe with any \( L_1 \), \( L_2 \), \( L_3 \), \( R_1 \), \( R_2 \), \( R_3 \), and still get the same tree, but just interchanged shoes. Therefore, we just multiply the tree's branches by 6, to get \( 6 \cdot 10 = \) <math>\boxed{\textbf{(A) } 60}</math>. | ||
+ | |||
+ | ~Pinotation | ||
+ | |||
+ | ~Diagram by Pinotation | ||
+ | |||
+ | ==Solution 3 (two cases)== | ||
+ | We have two main cases: | ||
+ | |||
+ | '''Case 1:''' <math>LLLRRR</math> or <math>RRRLLL</math> | ||
+ | |||
+ | There are <math>3! = 6</math> ways to assign the shoes to the ordering. For now we'll focus on the <math>LLLRRR</math> case and multiply by <math>2</math> since they're symmetrical. WLOG say we have <math>A_LB_LC_LC_RA_RB_R</math>. We can flip <math>A_L</math> and <math>B_L</math> among themselves and <math>A_R</math> and <math>B_R</math> among themselves, so we get <math>6 \cdot 2 \cdot 2 = 24.</math> Multiplying by <math>2</math> for symmetry gives us <math>48</math> ways for this case. | ||
+ | |||
+ | |||
+ | '''Case 2:''' <math>RRLLLR</math> or <math>RLLLRR</math> | ||
+ | |||
+ | Again, we'll focus on the <math>RRLLLR</math> case and multiply by <math>2</math> since they're symmetrical. WLOG say we have <math>B_RA_RA_LB_LC_LC_R</math>. We can assign the shoes to the ordering in <math>3! = 6</math> ways. <math>B_R</math> can also be moved over to be next to <math>C_R</math>. Then we would have <math>A_RA_LB_LC_LC_RB_R</math>, which is also valid. So this case gives us <math>6 \cdot 2 = 12</math> ways. | ||
+ | |||
+ | The answer is <math>48 + 12 = \boxed{60}.</math> | ||
+ | |||
+ | |||
+ | ~[[User:grogg007|grogg007]], ~EaZ_Shadow | ||
+ | |||
+ | ==Solution 4(focus on restrictions)== | ||
Notice that you cannot have <math>LRL</math> or <math>RLR</math> in a row, since you are guaranteed an <math>R</math> and an <math>L</math> from a different pair. This means you can either have three <math>L</math>'s in a row, three <math>R</math>'s in a row, or you have two <math>R</math>'s between two <math>L</math>'s and two <math>L</math>'s between two <math>R</math>'s. | Notice that you cannot have <math>LRL</math> or <math>RLR</math> in a row, since you are guaranteed an <math>R</math> and an <math>L</math> from a different pair. This means you can either have three <math>L</math>'s in a row, three <math>R</math>'s in a row, or you have two <math>R</math>'s between two <math>L</math>'s and two <math>L</math>'s between two <math>R</math>'s. | ||
Line 68: | Line 94: | ||
\end{align*} | \end{align*} | ||
− | We have <math>2\cdot 12+6\cdot 6=60.</math> | + | We have <math>2\cdot 12+6\cdot 6=\boxed{\textbf{(A) }60}.</math> |
~nevergonnagiveup | ~nevergonnagiveup |
Latest revision as of 23:33, 23 September 2025
Contents
Problem
Three different pairs of shoes are placed in a row so that no left shoe is next to a right shoe from a different pair. In how many ways can these six shoes be lined up?
Solution 1
Let denote the shoes.
There are ways to choose the first shoe. WLOG, assume it is
. We have
__, __, __, __, __.
Case
: The next shoe in line is
. We have
__, __, __, __. Now, the next shoe in line must be either
or
. There are
ways to choose which one, but assume WLOG that it is
. We have
__, __, __.
Subcase
: The next shoe in line is
. We have
__, __. The only way to finish is
.
Subcase
: The next shoe in line is
. We have
__, __. The only way to finish is
.
In total, this case has
orderings.
Case
: The next shoe in line is either
or
. There are
ways to choose which one, but assume WLOG that it is
. We have
__, __, __, __.
Subcase
: The next shoe is
. We have
__, __, __.
Sub-subcase
: The next shoe in line is
. We have
__, __. The only way to finish is
.
Sub-subcase
: The next shoe in line is
. We have
__, __. The remaining shoes are
and
, but these shoes cannot be next to each other, so this sub-subcase is impossible.
Subcase
: The next shoe is
. We have
__, __, __. The next shoe in line must be
, so we have
__, __. There are
ways to finish, which are
and
.
In total, this case has
orderings.
Our final answer is
Solution 2 (Tree Diagram)
Label the pairs \( (L_1, R_1) \), \( (L_2, R_2) \), and \( (L_3, R_3) \).
We now create a tree diagram. WLOG, we say our first tree diagram starts with \( L_1 \).
This tree has 10 branches.
Notice how we can interchange the first shoe with any \( L_1 \), \( L_2 \), \( L_3 \), \( R_1 \), \( R_2 \), \( R_3 \), and still get the same tree, but just interchanged shoes. Therefore, we just multiply the tree's branches by 6, to get \( 6 \cdot 10 = \) .
~Pinotation
~Diagram by Pinotation
Solution 3 (two cases)
We have two main cases:
Case 1: or
There are ways to assign the shoes to the ordering. For now we'll focus on the
case and multiply by
since they're symmetrical. WLOG say we have
. We can flip
and
among themselves and
and
among themselves, so we get
Multiplying by
for symmetry gives us
ways for this case.
Case 2: or
Again, we'll focus on the case and multiply by
since they're symmetrical. WLOG say we have
. We can assign the shoes to the ordering in
ways.
can also be moved over to be next to
. Then we would have
, which is also valid. So this case gives us
ways.
The answer is
~grogg007, ~EaZ_Shadow
Solution 4(focus on restrictions)
Notice that you cannot have or
in a row, since you are guaranteed an
and an
from a different pair. This means you can either have three
's in a row, three
's in a row, or you have two
's between two
's and two
's between two
's.
Below are the cases(note that once an is fixed the
adjacent to it is also fixed due to the constraint):
\begin{align*}
LLLRRR \Rightarrow 3!\cdot 2!=12\\
RLLLRR \Rightarrow 3!=6\\
RRLLLR \Rightarrow 3!=6\\
RRRLLL \Rightarrow 3!\cdot 2!=12\\
LRRRLL \Rightarrow 3!=6\\
LLRRRL \Rightarrow 3!=6\\
LRRLLR \Rightarrow 3!=6\\
RLLRRL \Rightarrow 3!=6\\
\end{align*}
We have
~nevergonnagiveup
Video Solution 1 by Pi Academy (Fast and Easy ⚡🚀)
https://youtu.be/c6nhclB5V1w?feature=shared
~ Pi Academy
Video Solution 2 by SpreadTheMathLove
https://www.youtube.com/watch?v=yYpnHoTQNi4
See also
2024 AMC 10B (Problems • Answer Key • Resources) | ||
Preceded by Problem 19 |
Followed by Problem 21 | |
1 • 2 • 3 • 4 • 5 • 6 • 7 • 8 • 9 • 10 • 11 • 12 • 13 • 14 • 15 • 16 • 17 • 18 • 19 • 20 • 21 • 22 • 23 • 24 • 25 | ||
All AMC 10 Problems and Solutions |
These problems are copyrighted © by the Mathematical Association of America, as part of the American Mathematics Competitions.